Ditrama

This topic has expert replies
User avatar
Senior | Next Rank: 100 Posts
Posts: 68
Joined: Fri Dec 03, 2010 10:29 pm
Thanked: 5 times
Followed by:1 members

Ditrama

by Bek » Thu Sep 22, 2011 2:19 pm
I need explanation for answer-choices C), D) and E)

The Source is PowerScore CR Bible
Attachments
Untitled.jpg

User avatar
Legendary Member
Posts: 1309
Joined: Mon Apr 04, 2011 5:34 am
Location: India
Thanked: 310 times
Followed by:123 members
GMAT Score:750

by cans » Thu Sep 22, 2011 7:06 pm
premise: each region receives share in proportion to total population share...
Premise: % revenue received were decreased for Korva, but population increased...
conclusion?

A)doesn't matter.
B) it can be possible that population grew by larger %
C) let M and G each had x people and each's population increased by y% each.
and K initially had p and increased by q%. where y>q
now initial share = p/(p+2x)
new share = 100p+pq/ (100p+pq +200x+2xy)
new share - initial share = 100p+pq/ (100p+pq +200x+2xy) - P/(p+2x)
100pp + 2pqx - 100pp -ppq - 200xp -2pxy / denominator = 2px(q-y) - ppq - 200xp /D
q-y<0 and thus numerator <0 and thus new share < initial share

Thus c) is the answer. it will explain why its share decreased...


D) let k initial=10, final=11,
m,g: 10000 and final 10002
thus initial: 10/20010= 4.9975 and final: 11/20015=5.49
thus even if numerical increase was smallest, share increased. thus not valid

E)say m's population doubled and k's increased by 90% and g's only 1%... still k can get more share..

IMO C
If my post helped you- let me know by pushing the thanks button ;)

Contact me about long distance tutoring!
[email protected]

Cans!!

User avatar
Senior | Next Rank: 100 Posts
Posts: 68
Joined: Fri Dec 03, 2010 10:29 pm
Thanked: 5 times
Followed by:1 members

by Bek » Thu Sep 22, 2011 8:30 pm
Initially, I also chose C, but it the correct answer is E.
Attachments
Untitled.jpg

Master | Next Rank: 500 Posts
Posts: 416
Joined: Thu Jul 28, 2011 12:48 am
Thanked: 28 times
Followed by:6 members

by gunjan1208 » Thu Sep 22, 2011 8:54 pm
Wow! Wow! Wow!

I loved the question trap. (I was also trapped by C)

I forgot to see that in the Stimulus it only talk about the decrease in Korava only. It does not talk about how other two countries id grow. That means there is a possibility that Koravacould stand in the middle also.

Now E makes sense.

That is why I have started to read POWERSCORE two days back:) I have heard its a very good source to learn reasoning.

Legendary Member
Posts: 2789
Joined: Tue Jul 26, 2011 12:19 am
Location: Chennai, India
Thanked: 206 times
Followed by:43 members
GMAT Score:640

by GmatKiss » Fri Sep 23, 2011 10:30 pm
Good one!! Again was trapped with C..

-GK

Master | Next Rank: 500 Posts
Posts: 222
Joined: Mon Oct 13, 2008 4:04 pm
Thanked: 3 times
Followed by:2 members

by venmic » Sat Sep 24, 2011 11:00 am
awesome post.. very good question

Thanks....

Bek wrote:I need explanation for answer-choices C), D) and E)

The Source is PowerScore CR Bi


ble

User avatar
GMAT Instructor
Posts: 2193
Joined: Mon Feb 22, 2010 6:30 pm
Location: Vermont and Boston, MA
Thanked: 1186 times
Followed by:512 members
GMAT Score:770

by David@VeritasPrep » Sat Sep 24, 2011 6:10 pm
This is indeed a good question. It is an LSAT question. It is from the December 1995 test, Logical reasoning section 2, # 24.

This is a good example of what I call "LSAT Math" there are a series of questions on the LSAT that deal with the difference between actual numbers and percentages. In this case, the actual number of citizens increased for KORVA. So that means simply that Korva/ Korva + Mitro + Guadar has to have decreased. The conclusion is that the numerator increased, but not by as much as the denominator.

It is not necessary that both M and G have increased by more than Korva only that one of them needs to have.

Another LSAT trick you can use is this: If answer A makes another answer correct you cannot choose A. In this case answer choice C makes choice E correct. Do you see it? Choice C says "The populations of Mitro and Guadar each increased by a percentage that exceeded the percentage by which the population of Korva increased." Well if it is true that they each increased more than Korva well choice E would then be forced to be correct. E "Korva's population grew by a smaller percentage than did the population of at least one of the other two autonomous regions."

Of course you are taking the GMAT and so this question will not translate exactly but you can learn from it. Anyway, it is a fun question I teach it a couple of times a year when I teach the LSAT.
Veritas Prep | GMAT Instructor

Veritas Prep Reviews
Save $100 off any live Veritas Prep GMAT Course